Tuesday, August 30, 2016

Mathematically Proving the Solution to, $ \frac{3^2+1}{3^2-1} + \frac{5^2+1}{5^2-1} + \frac{7^2+1}{7^2-1} + ..... + \frac{99^2+1}{99^2-1} $ and Other Strategies


TODO:
1. Reorganize. 2. Check for errors.

Initial Ideas:

Based on the level of expertise, this problem may be very hard or easy to solve. In this post I will write about my thinking process to solve this problem. Since this is a very long post, please report me if any mistakes are found.

By looking at the series above follows a pattern, where, the next term is an odd number. We can see the first term starts with 3, ignoring the $ +1, -1 $ for now, since they are constants in each term.
The next term can be calculated by,
current term variable = previous term variable + 2
So, after expanding the series above with a few more term,
$ \frac{3^2+1}{3^2-1} + \frac{5^2+1}{5^2-1} + \frac{7^2+1}{7^2-1} + \frac{9^2+1}{9^2-1} + \frac{11^2+1}{11^2-1} + ..... + \frac{97^2+1}{97^2-1} + \frac{99^2+1}{99^2-1} $

Approximating by Calculator and Generalization:

We can easily simply the series a little further and get a rough estimate on the possible solution. Simplifying the series,

$ \ \ \ \ \frac{3^2+1}{3^2-1} + \frac{5^2+1}{5^2-1} + \frac{7^2+1}{7^2-1} + \frac{9^2+1}{9^2-1} + \frac{11^2+1}{11^2-1} + ..... + \frac{97^2+1}{97^2-1} + \frac{99^2+1}{99^2-1} $

$ = \frac{9+1}{9-1} + \frac{25+1}{25-1} + \frac{49+1}{49-1} + \frac{81+1}{81-1} + \frac{121+1}{121-1} + ..... + \frac{9409+1}{9409-1} + \frac{9801+1}{9801-1} $

$ = \frac{10}{8} + \frac{26}{24} + \frac{50}{48} + \frac{82}{80} + \frac{122}{120} + ..... + \frac{9410}{9408} + \frac{9802}{9800} $

$ \approx 1.25 + 1.083 + 1.0416 + 1.025 + 1.016 + ..... + 1.00021 + 1.0002 $

$ \approx 49.5 \ \ \ \ (\text{Here}, \ \approx \text{ means approximately}) $

Note: This is not the true answer. I will show the original answer below, first by using a computer program, then proving mathematically.

About the calculation above:

By looking at the varying number above we can see the set is,
$ S = \{ n \ | \ n \text{ is an odd positive integer less than } 100 \} - \{1\} $

There are 100 positive integers between 1 to 100 including even and odd. So, there are half the odd numbers between 1 to 100 which is 50. The odd number set is,
$ S = \{ 1, 3, 5, 7, 11, 13, 15, .... , 95, 97, 99 \} $

We can clearly see the series above does not have 1 in it. If it did, then that could result is $ \frac{1^2+1}{1^2-1} = \frac{2}{0} $. The set can not contain 1 and we need to exclude it. So the set contains 49 positive odd integers from 3 to 99,
$ S = \{ 3, 5, 7, 11, 13, 15, .... , 95, 97, 99 \} $

Relation between set Members and Index:

$ S = \{ 1_1, 3_2, 5_3, 7_4, 11_5, 13_6, 15_7, .... , 95_{48}, 97_{49}, 99_{50} \} $

Here, the subscripts represents indices of set elements. Now, that 1 can not be a member of the set. Subtract all the indices by 1 and remove 1 element.
$ S = \{ 3_{2-1}, 5_{3-1}, 7_{4-1}, 11_{5-1}, 13_{6-1}, 15_{7-1}, .... , 95_{48-1}, 97_{49-1}, 99_{50-1} \} $

$ S = \{ 3_1, 5_2, 7_3, 11_4, 13_5, 15_6, .... , 95_{47}, 97_{48}, 99_{49} \} $

Relation between set index and the element at that index,
$ S_1 = 1 * 2 + 1 = 3 $
$ S_2 = 2 * 2 + 1 = 5 $
$ S_3 = 3 * 2 + 1 = 7 $
$...$
$...$
$...$
$ S_j = j * 2 + 1 = 2j + 1 $

So, the index of 99th term,
$2j + 1 = 99 $
$2j = 98 $
$j = 49 $


Format of the Series:

$ = \frac{10}{8} + \frac{26}{24} + \frac{50}{48} + \frac{82}{80} + \frac{122}{120} + ..... + \frac{9410}{9408} + \frac{9802}{9800} $
Each term of the sequence is, $ \{ \frac{m+2}{m} \ | \ m \in \mathbb{Z^+} \wedge m \text{ is odd } \wedge m < 100 \} $.

Expressing the Series as Summation:

$ \frac{3^2+1}{3^2-1} + \frac{5^2+1}{5^2-1} + \frac{7^2+1}{7^2-1} + ..... + \frac{99^2+1}{99^2-1} $

I can rewrite the series above as,
$ \frac{S_1^2+1}{S_1^2-1} + \frac{S_2^2+1}{S_2^2-1} + \frac{S_3^2+1}{S_3^2-1} + ..... + \frac{S_4^2+1}{S_4^2-1} $

Expressing it as Summation,
$ \sum\limits_{i=1}^{49} \frac{S_i^2+1}{S_i^2-1} $

or, write is as,
$ \sum\limits_{\substack{3 \leq i \leq 99 \\ i + 2}} \frac{i^2+1}{i^2-1} $

Converting the Series Summation to Code:

I will use $ \sum\limits_{\substack{3 \leq i \leq 99 \\ i + 2}} \frac{i^2+1}{i^2-1} $ representation to convert it into code. It can be very easily translated into code. Look at the diagram below for the grouping,



For loop starts from initial point and loops until its condition is reached and each time it increments by a certain value. Other loop types can be also implemented. Now this grouping can be translated to a loop like this,




C++ Code:

Output of the code above:
49.490000



Patterns in the Series as Sum of Another:




Why the last one is $ 392 $? We can calculate the value to sum by looking at the diagram above.
$ \text{value to sum} = \text{ index of the next term denominator} * 8 $.

So, we can get the next term using current value to sum. All the numerator values are multiple of $ 8 $ and the difference of two immediate numerators are also multiple of $ 8 $, which are same above and below.
$ numerator = denominator + 2 $

$ \text{next term denominator} = \text{current term denominator} + \text{value to sum} $
$ \text{next term denominator} = \text{current term denominator} + \text{index of the next term} * 8 $

So, $ \frac{7^2+1}{7^2-1} = \frac{50}{48} $ it depends on $ \frac{5^2+1}{5^2-1} = \frac{26}{24} $. $ \frac{7^2+1}{7^2-1} $ is in the $ 3rd $ position of the sequence set. Using the formula above,

$ \text{next term denominator} = 24 + 3 * 8 = 48 $ and $ numerator = denominator + 2 = 48 + 2 = 50 $
This is same as $ \frac{7^2+1}{7^2-1} = \frac{50}{48} $. Similarly, we can get the sum value for the last term denominator and numerator. We know $ \frac{99^2+1}{99^2-1} $ is in the $ 49 $ position or, index.

$ \text{value to sum} = \text{ index of the next term denominator} * 8 = 49 * 8 = 392 $. If the next term denominator is 9800 and value to sum is 392, we can get the, $ \text{previous term denominator} = 9800 - 392 = 9408 $ and the $ numerator = 9408 + 2 = 9410 $. We get 9800, when the variable n is $ 49 $, so we are trying to calculate the denominator and numerator of its precious term which has index $ 48 $.

In the $ 48th $ index, $ n = 97 $, so, $ \frac{97^2+1}{97^2-1} = \frac{9410}{9408} $. They are a match. Using the technique above we can get the any other term based on another term.

Patterns in the Series as Multiple of Another:


Each $ denominator $ is multiple of and each $ numerator - 2 $ is also multiple of 2. There are 49 terms in the series. So, there are 48 plus signs in between. Ex: We can partition two things with one symbol, $ \Box \ | \ \Box $. Similarly, we can partition two things with one symbol, $ \Box \ | \ \Box | \ \Box $. so, if there are $ m $ things we need $ m - 1 $ symbols to partition.

There are 48 multiples of 8 to generate the next terms from the first term. Each time they are incremented by 1 starting from 3. So the progression is,
$ 3 + 4 + 5 + 6 + 7 + ..... + 49 $

We know sum from 1 to n is, $ \sum_{i=1}^{n} i = \frac{n*(n+1)}{2} $. Get the multiple of 8 denominator using the index in the set.
$ denominator = \frac{index*(index+1)}{2} * 8 $. For, the last one( 49th term ) $ index = 49 $, $ denominator = \frac{49*(49+1)}{2} * 8 = 1225 * 8 = 9800 $. We can also get the previous term multiple by, $ \text{next term multiple} = \text{sum to get next term} + ( \text{current term denominator} / 8 ) $.

The previous term is $ \frac{97^2+1}{97^2-1} $, where, $ n = 48 $.
$ ( \text{next term multiple} - \text{sum to get next term} ) * 8 = \text{current term denominator} $
$ \text{current term denominator} = ( 1225 - 49 ) * 8 = 9408 $. Also the denominator of $ \frac{97^2+1}{97^2-1} $ is $ \frac{9410}{9408} $.

Pattern in the series $ 1 + \frac{1}{3} + \frac{1}{6} + \frac{1}{10} + \frac{1}{15} + \frac{1}{21} + ..... + \frac{1}{1225}: $


Solving the Series $ 1 + \frac{1}{3} + \frac{1}{6} + \frac{1}{10} + \frac{1}{15} + \frac{1}{21} + ..... + \frac{1}{1225}: $

So far we have,
$ 49 + \frac{1}{4} * \Bigg( 1 + \frac{1}{3} + \frac{1}{6} + \frac{1}{10} + \frac{1}{15} + \frac{1}{21} + ..... + \frac{1}{1225} \Bigg) $
Since the true answer is 49.49, we can deduce that,
$ \frac{1}{4} * \Bigg( 1 + \frac{1}{3} + \frac{1}{6} + \frac{1}{10} + \frac{1}{15} + \frac{1}{21} + ..... + \frac{1}{1225} \Bigg) = 0.49 $
So, the sequence above should result in,
$ 1 + \frac{1}{3} + \frac{1}{6} + \frac{1}{10} + \frac{1}{15} + \frac{1}{21} + ..... + \frac{1}{1225} = 1.96 $
At first glance this looks very hard, but it too can be calculated mathematically. The Sequence set of the above series is,
$ P = \{ 1, \frac{1}{3}, \frac{1}{6}, \frac{1}{10}, \frac{1}{15}, \frac{1}{21}, ....., \frac{1}{1225} \} $
Relation between set index and elements,
$ P = \{ P_1, P_2, P_3, P_4, P_5, P_6, ....., P_{49} \} $
$ P_1 = \frac{2}{1*(1+1)} = 1 $

$ P_2 = \frac{2}{2*(2+1)} = \frac{1}{3} $

$ P_3 = \frac{2}{3*(3+1)} = \frac{1}{6} $

$ P_4 = \frac{2}{4*(4+1)} = \frac{1}{10} $

$ P_5 = \frac{2}{5*(5+1)} = \frac{1}{15} $

$ P_6 = \frac{2}{6*(6+1)} = \frac{1}{21} $
$ ... $
$ ... $
$ ... $
$ P_{49} = \frac{2}{49*(49+1)} = \frac{1}{1225} $

So, we can calculate the $ i $ th member of $ P $ set using formula,
$ P_i = \frac{2}{i*(i+1)} $

The summation can be expressed as,
$ S_P = \sum_{i=1}^{49} \frac{2}{i*(i+1)} $



The problem still isn't solved yet, since it is summation from and still needs to be calculated. Now express the terms in summation in such a way that some terms are canceled and we are left with a simple form.
We can represent $ \frac{2}{i*(i+1)} $ as,
$ \frac{2}{i*(i+1)} \equiv \frac{A}{i} + \frac{B}{i+1} $
Now, multiply $ A $ with $ (i+1) $ and $ B $ with $ Bi $ which equals 2, since our numerator have be 2 to keep them equivalent.
$ 2 = A*(i+1) + B*i $

To calculate value of $ A $ and $ B $, choose i in a way such that the other one becomes zero,
if, $ i = 0 $, $ 2 = A*(0+1) + B*0, So, A = 2 $

if, $ i = -1 $, $ 2 = A*(-1+1) + B*(-1), So, B = -2 $

Now we can write the original equation as, $ \frac{2}{i*(i+1)} = \frac{2}{i} + \frac{-2}{i+1} $

$ \frac{2}{i*(i+1)} = 2 * \Bigg( \frac{1}{i} - \frac{1}{i+1} \Bigg) $

The sum of the sequence from $ i = 1 \text{ to } n $ is,

$ \sum_{i=1}^{n} \frac{2}{i*(i*1)} = \sum_{i=1}^{n} \frac{2}{i} + \frac{-2}{i+1} $

$ \sum_{i=1}^{n} \frac{2}{i*(i*1)} = 2 * \sum_{i=1}^{n} \Bigg( \frac{1}{i} - \frac{1}{i+1} \Bigg) $

Here, $ n = 49 $ since we need to capitulate the sum of 49 terms,

$ \sum_{i=1}^{49} \Bigg( \frac{1}{i} - \frac{1}{i+1} \Bigg) $

Expanding the summation,

$ \require{cancel} 2 * \sum_{i=1}^{n} \Bigg( \frac{1}{i} - \frac{1}{i+1} \Bigg) = 2 * [ ( \frac{1}{1} - \cancel{\frac{1}{2}} ) + ( \cancel{\frac{1}{2}} - \cancel{\frac{1}{3}} ) + ( \cancel{\frac{1}{3}} - \cancel{\frac{1}{4}} ) + ( \cancel{\frac{1}{4}} - \cancel{\frac{1}{5}} ) + ..... + ( \cancel{\frac{1}{n-1}} - \cancel{\frac{1}{n}} ) + ( \cancel{\frac{1}{n}} - \frac{1}{n+1} ) ] $

$ 2 * \sum_{i=1}^{n} \Bigg( \frac{1}{i} - \frac{1}{i+1} \Bigg) = 2 * [ 1 - \frac{1}{1+n} ] $

So, $ \sum_{i=1}^{n} \frac{2}{i*(i*1)} = 2 * \sum_{i=1}^{n} \Bigg( \frac{1}{i} - \frac{1}{i+1} \Bigg) = \frac{2n+2-2}{1+n} = \frac{2n}{1+n} $

Recall that, $ \sum_{i=1}^{n} \frac{2}{i*(i*1)} $ represents sum of series $ 1 + \frac{1}{3} + \frac{1}{6} + \frac{1}{10} + \frac{1}{15} + \frac{1}{21} + ..... + \frac{1}{1225} $, where, $ n = 49 $.

$ 1 + \frac{1}{3} + \frac{1}{6} + \frac{1}{10} + \frac{1}{15} + \frac{1}{21} + ..... + \frac{1}{1225} = \frac{2n}{1+n}, \text{ where, n = 49} $

$ 1 + \frac{1}{3} + \frac{1}{6} + \frac{1}{10} + \frac{1}{15} + \frac{1}{21} + ..... + \frac{1}{1225} = \frac{2*49}{1+49} $

$ 1 + \frac{1}{3} + \frac{1}{6} + \frac{1}{10} + \frac{1}{15} + \frac{1}{21} + ..... + \frac{1}{1225} = \frac{98}{50} $

$ 1 + \frac{1}{3} + \frac{1}{6} + \frac{1}{10} + \frac{1}{15} + \frac{1}{21} + ..... + \frac{1}{1225} = 1.96 \ \ \ \ \Box $

Results from Wolfram Alpha:

Input:
Sum[(k^2+1)/(k^2-1),{k,3,99,2}]

Alternatively, follow the link below to get the result.
https://www.wolframalpha.com/input/?i=Sum%5B(k%5E2%2B1)%2F(k%5E2-1),%7Bk,3,99,2%7D%5D

Solution to the Problem:

$ \ \ \ \ \frac{3^2+1}{3^2-1} + \frac{5^2+1}{5^2-1} + \frac{7^2+1}{7^2-1} + \frac{9^2+1}{9^2-1} + ..... + \frac{99^2+1}{99^2-1} $

$ = \frac{10}{8} + \frac{26}{24} + \frac{50}{48} + \frac{82}{80} + ..... + \frac{9802}{9800} $

$ = \frac{8+2}{8} + \frac{24+2}{24} + \frac{48+2}{48} + \frac{80+2}{80} + ..... + \frac{9800+2}{9800} $

$ = \frac{8}{8} + \frac{2}{8} + \frac{24}{24} + \frac{2}{24} + \frac{48}{48} + \frac{2}{48} + \frac{80}{80} + \frac{2}{80} + ..... + \frac{9800}{9800} + \frac{2}{9800} $

$ = 1 + \frac{2}{8} + 1 + \frac{2}{24} + 1 + \frac{2}{48} + 1 + \frac{2}{80} + ..... + 1 + \frac{2}{9800} $

$ = 49 + \frac{2}{8} + \frac{2}{24} + \frac{2}{48} + \frac{2}{80} + ..... + \frac{2}{9800} $

$ = 49 + \frac{1}{4} + \frac{1}{12} + \frac{1}{24} + \frac{1}{40} + ..... + \frac{1}{4900} $

$ = 49 + \frac{1}{4} * \Bigg( 1 + \frac{1}{3} + \frac{1}{6} + \frac{1}{10} + \frac{1}{15} + \frac{1}{21} + ..... + \frac{1}{1225} \Bigg) $

$ = 49 + \frac{1}{4} * \frac{2*49}{49+1} $

$ = 49 + \frac{1}{4} * 1.96 $

$ = 49.49 \ \ \ \ \Box $


No comments: